Questions tagged [law-of-large-numbers]

For questions about the law of large numbers, a classical limit theorem in probability about the asymptotic behavior (in almost sure or in probability) of the average of random variables. To be used with the tags (tag:probability-theory) and (tag: limit-theorems).

Filter by
Sorted by
Tagged with
1 vote
0 answers
20 views

Confusion about LLN for Empirical Processes/Measures

I have seen the following LLN result on empirical measures: (Statement 1) Let $X, X_1, X_2, \cdots, X_n$ be i.i.d. random variables taking values in $[0, 1]$. Then $$ \mathbb{E}\sup_{f \in \mathcal{F}...
Partial T's user avatar
  • 551
0 votes
0 answers
43 views

"Elementary" Proof of Strong Law of Large Numbers

I came across a paper that a seemingly "elementary" proof of the strong law of large numbers, although I am having a lot of trouble understanding it since the author leaves a lot of ...
Aadi Rane's user avatar
  • 291
1 vote
0 answers
39 views

If $\sup_{f\in F} \Big|||f||_n-||f|| \Big|\overset{p}{\to} 0$ then $\sup_{f\in F, ||f||\neq 0} \Big| ||f||_n/||f||-1 \Big| \overset{p}{\to} 0$.

SETUP: Let $\{X\}_{t\in\mathbb{N}}$ be a strictly stationary stochastic process on the probability space $(\Omega, \mathcal{F}, P)$ where for each $t\in\mathbb{N}$ the random elements are such that $...
Chad Brown's user avatar
2 votes
1 answer
61 views

How can an estimator be consistent and asymptotically normal at the same time?

I can't work out why the asymptotic distribution of an estimator matters if it is consistent? My understanding is: An estimator, $\hat{\theta}$, is consistent if it converges in probability to the ...
arb6's user avatar
  • 23
1 vote
1 answer
108 views

The function $\log^+x=\max\{1, \log x\}$.

I was reading Marcinkiewicz-Zygmund (MZ) law of large numbers for random fields and came across necessary and sufficient condition $E(|X|\log^+|X|)< \infty$ for MZ-SSLN to hold true. I have a ...
Shyam's user avatar
  • 31
2 votes
1 answer
212 views

Law of large number with subset of the variables

Let $(X_i, Y_i)_{i=1}^{\infty}$ be iid continuous random vectors with continuous joint density, where $X_1$ have support $\mathcal{X}$. Let $B_n\subset \mathcal{X}\subset\mathbb{R}$ be decreasing ...
Albert Paradek's user avatar
5 votes
0 answers
84 views

What is the Asymptotic Order of the Sum of Random Variables with non-finite Moments?

Suppose $X_i$ is independently and identically distributed over $i$, and $E(|X_i|^d)$ with $d>0$ is not finite or undefined: I am wondering whether $\sum_{i=1}^N |X_i|^d$ is of any asymptotic ...
Jack's user avatar
  • 121
0 votes
0 answers
50 views

For a random binary sequence of length $2N$ with $n$ 1's, what is the probability that the first $N$ digits have a 'similar' proportion of 1's?

More precisely, if we know the proportion of 1's in a uniformly randomly chosen binary sequence of length $2N$ is $p$, then given an $\epsilon>0$ , what is the probability that the proportion of 1'...
Beatnik Dopa's user avatar
1 vote
1 answer
29 views

Understanding a step in this proof of the $L^4$-SLLN

Let $(X_n)_{n \ge 1}$ be independent with $E(X_k) = \mu$ and $E((X_k)^4) \le C$ for some $C>0$. Prove that if $S_n = X_1 + ... + X_n$ then $\frac{S_n}{n} \to \mu$ almost surely as $n \to \infty$. ...
Ryderr's user avatar
  • 125
0 votes
0 answers
32 views

Difference of hypotheses between weak and strong law of large numbers

I have read several questions on the difference between weak and strong laws in terms of convergence in probability vs convergence a.s. and I think I grasp them. However, I have uncertainty on the ...
MysteryGuy's user avatar
0 votes
0 answers
31 views

A lemma involving Chebyshev's inequality to prove the weak law of large numbers

Reading Roch's Modern Discrete Probability one finds the following exercise (2.6): (Sums of uncorrelated variables). Centered random variables $X_1,\dots X_n$ are uncorrelated if forr all $r$ and for ...
René Quijada's user avatar
1 vote
0 answers
22 views

Monotonicity of the probability of a sum of independent random variables being below a threshold

Suppose I have a sequence of i.i.d. random variables $X_1, X_2, \dots$ with positive mean $E[X_1] = \mu$. For $A>0$, is the function $$f(n) = \Pr( X_1 + X_2 + \dots + X_n \leq A) $$ monotonically ...
ilanshom's user avatar
1 vote
0 answers
25 views

Central Limit Theorem for Difference-in-Means Estimator

I am studying Lecture 1 of Stefan Wager's Causal Inference notes and come across a central limit theorem for the difference-in-means estimator, which I am unable to prove. The mathematical abstraction ...
Kittayo's user avatar
  • 699
0 votes
1 answer
50 views

Consistency of Biased Estimators

In Statistical Inference, we were taught this theorem, Consider an estimator $T_n$ of population parameter $\theta$, using $n$ samples. $T_n$ is a Consistent Estimator of $\theta$ if $$E[T_n] \to \...
Harry's user avatar
  • 21
1 vote
2 answers
148 views

Bound on expected norm of the difference between the sample mean $\bar{X_n}$ and population mean $\mu$ as a function of the sample size $n$ for LLN?

My question is motivated by this question: Does law of large numbers converge in $L^1$? that asks about the the convergence in $L^1$-norm of the sample mean $\bar{X_n}$ to the population mean $\mu.$ I ...
Learning Math's user avatar
2 votes
0 answers
39 views

Application of ergodic theorem to transformed Gaussian process

Assume that $W$ is a zero-mean Gaussian process on $[0,\infty)$ with covariance function satisfying $c(s,t):=E(W(s)W(T))=C(|t-s|)$ for a function $C:[0,\infty) \to \mathbb{R}$. Can I exploit somehow ...
Jack London's user avatar
  • 1,766
2 votes
0 answers
84 views

Law of large numbers for non-independent and non-identically distributed samples

Let $X \sim p_X$ be a real-valued random variable with $\mathbb{E}[X] = \mu > c$ where $c \in \mathbb{R}.$ Assume you sample from $p_X$ and only accept samples such that the current sample mean is ...
tobayes's user avatar
  • 21
1 vote
1 answer
29 views

Weak law of large numbers for distributed Bernoulli random variables in a particular case. [closed]

Let $X_1, X_2, ...$ be a sequence of independent and identically distributed random variables, such that $X_i\sim Bern(p)$. Now, let $Y_{1,n}, Y_{2,n}, ...$ be random variables, such that $Y_{i,n}\sim ...
Helder Alves Arruda's user avatar
0 votes
0 answers
26 views

Empirical Distribution Convergence, Ordering Of Samples

I am trying to formally justisty a "rearrangement" algorithm, which rearranges the samples of two random variables to reflect a certain joint distribution. Suppose that we have two pairs $(...
Nicola Zaugg's user avatar
3 votes
0 answers
76 views

Uniform law of large numbers for ergodic stationary sequence

I am trying to apply a uniform law of large numbers, which is stated in Lemma 7.2 of "Econometrics" by Fumio Hayashi. The starting point is the stochastic process $\{x_t\}$, which we assume ...
Kristan's user avatar
  • 81
0 votes
1 answer
30 views

How to understand the proof of the division theorem in the four arithmetic operation theorems of probability convergence for random variables?

\begin{equation} Y_n \stackrel{P}{\longrightarrow} b \text {} \end{equation} and: \begin{aligned} & P\left(\left|\frac{1}{Y_n}-\frac{1}{b}\right| \geqslant \varepsilon\right)=P\left(\left|\frac{...
SUNi's user avatar
  • 3
1 vote
0 answers
60 views

Suppose $(X_{n})_{n}$ is series of i.i.d. random variables and $E(X_{1})=2$. Compute $\lim_{n\to\infty} (n \ln (\varphi_{X_{1}}(\frac{2}{n})))$

Let's define $S_{n}=\sum_{k=1}^{n} X_{k}$. According to strong law of large numbers we have $\frac{S_{n}}{n} \rightarrow E(X_{1})=2$ a.s. It follows from that $\frac{S_{n}}{n} \rightarrow 2$ in ...
bnagy01's user avatar
  • 97
1 vote
0 answers
38 views

Inifinite numbers of i.i.d Markov chain and law of large number.

Let $(X(t))_{t \geq 0}$ be a continuous time-homogeneous Markov chain with values in the state space $S$ that we assume to be finite (countable should also work). We denote $P = (p_{ij})_{ij}$ the ...
Velobos's user avatar
  • 2,190
2 votes
1 answer
94 views

Quantitative law of large numbers for non-identically distributed random variables

I don't know much about probability so this may be an easy consequence of some well known theorem. Suppose we have a sequence of independent, $\mathbb{C}$-valued random variables $(\xi_n)_{n\in\...
Saúl RM's user avatar
  • 3,413
1 vote
2 answers
251 views

Law of Large Number for Stochastic Processes

Consider the following Stochastic Process: $$B(t)∼N(μt,σt)$$ Here is a simulation for multiple possible trajectories of this Stochastic Process (R Code): ...
stats_noob's user avatar
  • 3,176
1 vote
0 answers
42 views

How do I prove that the strong law of large numbers is equivalent to $P(∩_{n=1}^∞ ∪_{{k}\geq{n}}{|{\frac{1}{k}}S_k-p|\geq{ε}})=0$

I want to solve the following task: Show that the strong law of large numbers statement is equivalent to the following statement: For all ε>0: $$P(∩_{n=1}^∞∪_{{k}\geq{n}}{|{\frac{1}{k}}S_k-p|\geq{ε}...
MathStudentRUB's user avatar
2 votes
1 answer
87 views

Convergence of a capital describing random variable

In my previous question Infinite game of an unfair coin toss, I showed that in a game between Person A and B, where: Person A has an unfair coin with probability $p \in (\frac{1}{3},\frac{1}{2})$ of ...
tychonovs-scholar's user avatar
0 votes
0 answers
27 views

Weak Law of Large Numbers (Non-convergence in probability)

Given $X_1, X_2,..., X_n$ independent random variables with $P(X_n=4^n)=P(X_n=-4^n)=1/2$. Let $S_n = X_1 + X_2 +... + X_n$. Determine for which $\epsilon>0$, if any, that the $P(|S_n|/n>\epsilon)...
tt99999's user avatar
  • 11
0 votes
1 answer
62 views

Prove or Disprove Almost Sure Convergence

Let $𝑋_1,𝑋_2, … , 𝑋_𝑛$ be a sequence of random variables, with $𝐸(𝑋_𝑛) = 7$ and $𝑉𝑎𝑟(𝑋_𝑛) = 1/sqrt(𝑛^𝑎)$ for each 𝑛. Prove or disprove that ${𝑋_𝑛}$ must converge to 7 with probability ...
john22445's user avatar
1 vote
0 answers
74 views

Application of Weak Law of Large Number

Let $𝑋_1,𝑋_2, … , 𝑋_𝑛$ be a sequence of independent random variables with $𝑃(𝑋_𝑛 = 4^𝑛) = 𝑃(𝑋_𝑛 = −4^𝑛) = \frac12$. Let $𝑆_𝑛 = 𝑋_1 + 𝑋_2 + ⋯ + 𝑋_𝑛$. Determine for which $𝜀 > 0$, ...
john22445's user avatar
0 votes
1 answer
78 views

Converse of the weak law of large numbers [closed]

I solved a nice exercise which consists of proving a converse for the law of large numbers which is, if $(X_i)_{i \in\mathbb{N}^*}$ are iid random valued such that $$ \frac{X_1+\ldots+X_n}{n} \to Y$$ ...
Rafaël's user avatar
  • 181
3 votes
2 answers
155 views

Asymptotic convergence of a count-distinct estimator

Consider the following variant of the count-distinct problem. Problem Setting. Let $B = \{ b_1, \dots, b_n \}$ be a finite set of $n$ balls, let $C$ be a set of colours, let $F : B \to C$ be a ...
Boris's user avatar
  • 33
4 votes
2 answers
93 views

Strong law of large numbers for $\mathrm{Bin}(n, p_n)$ variables

Massive edit to simplify the central question Suppose $X_n\sim \mathrm{Bin}(n, p_n)$ be a collection of independent random variables such that $np_n\to \infty$. Can we say that $Y_n:=X_n/np_n\to 1$ ...
Landon Carter's user avatar
0 votes
0 answers
83 views

Sufficient Conditions for Strong Law of Large Numbers

In this paper, I found that if $\lbrace X_i \rbrace$ is an independenly indentically distributed (i.i.d.) sequence and $E(|X_i|)<\infty$, then $\bar{X}_{N}=\sum_{i=1}^N\frac{X_i}{N}$ converge to $E(...
Jack's user avatar
  • 121
0 votes
0 answers
69 views

sum of two point distribution converge to infinity almost surely

Question: $\{X_k\}_{k=1}^{\infty}$ i.i.d.,$P(X_1=-1)=P(X_1=1)=\frac12$. $S_n=\sum\limits_{k=1}^n X_k$. Prove that: 1.$S_n$ converge to infinity almost surely,i.e. $P(\lim_{n\to+\infty}S_n=\infty)=1$; ...
shdvt's user avatar
  • 619
2 votes
1 answer
88 views

Weak vs strong law of large numbers

Let $\{X_n\}_{n \in \mathbb{N}}$ be a sequence of real i.i.d. random variables with mean $\mu$. Let $S_n$ be the sum of the first $n$ elements of this seqeuence, $$S_n = \frac1n\sum_{i=1}^n X_i.$$ ...
CBBAM's user avatar
  • 5,975
1 vote
1 answer
126 views

Convergence in Probability (Law of Large Numbers)

Suppose $X_1,X_2,…,X_n$ are iid Poisson random variables, each with mean $\theta$. How to prove that $Y_n=\exp[−\frac{1}{n}(X_1+X_2+⋯+X_n)]$ converges in probability to $P(X=0)=\exp(−\theta)$ ? Hint: ...
john22445's user avatar
0 votes
1 answer
167 views

Convergence in Probability (Weak Law of Large Number)

Suppose $𝑌_1, 𝑌_2, … , 𝑌_𝑛$ are independent and identically distributed Poisson random variables, each with mean $𝜆$. Prove that $𝑋𝑛 =exp[−(1/𝑛)(𝑌_1 + 𝑌_2 + ⋯ + 𝑌_𝑛)]$ converges in ...
john22445's user avatar
4 votes
1 answer
109 views

Epsilon-Delta analysis for the Law of Large Numbers?

Law of Large Numbers: For a sequence of independent and identically distributed random variables $X_1, X_2, X_3, ..., X_n$, each with an expected value $E[X_i] = \mu$ and sample estimator $\overline{...
stats_noob's user avatar
  • 3,176
0 votes
0 answers
33 views

Computing the expectation of random variable by marginalizing over the condtional expectation and prove with LLN

I am trying to estimate the expectation of $X$ by marginalize over $Y$ and the conditional expectation $E(X|Y)$ and prove that it converges to the true expectation of $X$ as the sample size approaches ...
Coodyyy's user avatar
1 vote
1 answer
37 views

$\{X_k\}$ are iid (the standard normal distribution), what is the distribution of $\lim \tau_n=\frac{X_{n+1}}{\sqrt{\sum_{k=1}^n X_k^2/n}}$?

$\{X_k\}_{k=1}^{\infty}$ are independent and identically distributed (the standard normal distribution), $$Y_n^2=\sum_{k=1}^n X_k^2, \ \ \ \tau_n=\frac{X_{n+1}}{\sqrt{Y_n^2/n}}.$$ What is the ...
fragileradius's user avatar
0 votes
1 answer
39 views

Confusion on one step in proof in Achim Klenke's Probability Theory

I've come to the following theorem and proof in Klenke's Probability Theory book, and I've hit a roadblock on the very last step of the proof. We know $\limsup_{n\to\infty}|k_n^{-1}S_{k_n}-\mathbf{E}[...
modz's user avatar
  • 101
2 votes
1 answer
87 views

Proof of Marcinkiewicz-Zygmund strong law of large numbers

Marcinkiewicz-Zygmund strong law of large numbers: Let $X_1,X_2,···$ be i.i.d. with $E|X_1|^p< \infty$ for some $0< p <2$. Then $$ \begin{cases} \frac{S_n - nEX}{n^{1/p}} \to 0 \text{ a.s.} &...
Fireond's user avatar
  • 21
0 votes
0 answers
33 views

Seeking help with the application of Law of Large Numbers and Central Limit Theorem to calculate Investor Risk

I'm a newbie to the forum with zero financial or statistical skills - first time post...seeking some assistance and a solution..thanks in advance! I am trying to create a investor calculator or at ...
Darren's user avatar
  • 1
1 vote
0 answers
43 views

$\text{Var}(X_n) < \infty$ for all $n$, $\text{Var}(S_n) = o(n^2)$ when $n \to \infty.$ Prove $\frac{S_n}{n} \to E(S_n)$ in probability.

Random variables $X_n$ are independent, but not identically distributed. $\text{Var}(X_n) < \infty$ for all $n$, $\text{Var}(S_n) = o(n^2)$ when $n \to \infty.$ Prove $\frac{S_n - E(S_n)}{n} \to 0$ ...
fragileradius's user avatar
0 votes
1 answer
69 views

Step in the proof of the strong law of large numbers.

Let $(X_i)_{i \geq 1}$ i.i.d. and $\mathbb{E}(|X_1|)< \infty$, w.l.o.g. $\mathbb{E}(X_1) = 0$. This in turn implies: $$ \sum_{n=1}^{\infty} \mathbb{P}(|X_1| \geq n) \leq \int_0^{\infty} \mathbb{P}(|...
Henry T.'s user avatar
  • 1,297
2 votes
0 answers
70 views

Will the expanding cube look gray?

Consider the tiling of $\mathbb{R}^n$ by unit cubes centered at integer lattice points, i.e. $$ \mathbb{R}^n = \bigcup_{a \in \mathbb{Z}^n} Q\left(a, \frac{1}{2}\right). $$ Color each unit cube black ...
Cauchy's Sequence's user avatar
5 votes
1 answer
109 views

Strong Law of Large Numbers for increasing index sets

Let $(X_n)_{n \in \mathbb{N}}$ be a sequence of integrable i.i.d random variables. Let $I_n \subset \mathbb{N}$ satisfy $\lim_{n \to \infty} |I_n| = \infty$, where $|I_n|$ is the cardinality of $I_n$. ...
Cauchy's Sequence's user avatar
0 votes
0 answers
43 views

Law of large numbers for part sum of terms in random integer composition

Let $C=C_{n,m}=(c_1,\ldots,c_n)$ be a random weak $n$-part integer composition of $m$ drawn uniformly from all $\binom{m+n-1}m$ such compositions. Suppose $m\to\infty$ as $n\to\infty$ (e.g. $m\sim n^\...
David Bevan's user avatar
  • 5,861
1 vote
1 answer
114 views

The Money Left in an Infinite Gambling Game

Let $(\Omega, \mathcal{F}, \mathbb{P})$ be a probability space. Suppose that $\Omega$ is defined as below \begin{equation*} \Omega = \Big\{ \omega = (\omega_1, \omega_2, \dots) \big|\, \omega_i \in \{...
Hosein Rahnama's user avatar

1
2 3 4 5
20